CPAN Flashcards
A 64-year-old man presents for elective laparoscopic cholecystectomy under general anesthesia. The patient has a two-year history of Parkinson’s disease and is taking levodopa/carbidopa. Which of the following is the most appropriate prophylactic antiemetic for this patient?
- Metoclopramide
- Ondansetron
- Prochlorperazine
- Droperidol
The correct answer is:
- Ondansetron
Patients taking levodopa/carbidopa show a high incidence of postoperative nausea and vomiting. This is related to the direct activation of dopamine (D2) receptors by the levodopa in the chemoreceptor trigger zone.
Ondansetron has neither anti-dopaminergic activity nor any agonist action at acetylcholine receptors. It blocks the action of serotonin at 5-HT3 receptors to produce antiemesis.
Ondansetron is an ideal agent for postoperative nausea prophylaxis as it has no sedative potential.
Ondansetron efficacy is higher than metoclopramide for nausea prophylaxis.
A 52-year-old man is to undergo a routine open inguinal hernia repair. He has no past medical history and takes no regular medications. In the anesthetic room, he is given propofol, fentanyl, and succinylcholine intravenously. Absence of which of the following enzymes could prolong the neuromuscular blockade effects induced by one of the anesthetic drugs?
- Glucose-6-phosphate dehydrogenase
- Plasma cholinesterase
- Dopamine beta-hydroxyl add
- Cytochrome P450 oxidase
The correct answer is :
- Plasma cholinesterase
Succinylcholine chloride is a short acting depolarizing neuromuscular blockade that is approved by the United States food and drug administration (FDA) as a provision to other sedatives or hypnotics. It is a correlate acetylcholine (ACh); hence, it disrupts all cholinergic receptors of the parasympathetic and sympathetic nervous system.
Decreased levels of plasma cholinesterase are seen in patients with genetic defects, pregnancy malignancy, severe liver disease, chronic renal failure burns infections, anemia, myxedema, and peptic ulcer disease.
This can lead to prolonged effects of succinylcholine, including complete skeletal muscle paralysis. Hyperkalemia can also occur.
The activity of plasma cholinesterase can be decreased by the use of oral contraceptives, corticosteroids, MAO inhibitors, or organophosphate insecticides.
A 16-year-old male student presents to the emergency department after being assaulted by a drug dealer. The patient is conscious and gives a history of being stabbed in the abdomen. The primary survey reveals a 3cm linear deep wound in his epigastric region. His vitals are taken that show a heart rate of 118 beats/minute, a blood pressure of 85/55 mmHg, and a respiratory rate of 20/minute. The patient is resuscitated and shifted to operation room for urgent exploratory laparotomy. During the laparotomy, an anterior gastric perforation is visualized, which is then repaired using the Modified Graham patch repair technique. What is the method of choice for managing postoperative pain in this patient?
- As needed pain medications on demand
- Patient-controlled analgesia
- Epidural anesthesia
- Spinal anesthesia
The correct answer is:
- Patient-Control Analgesia (PCA)
In the hospital, patient-controlled analgesia (PCA) is the best way to deliver pain medication post-operatively.
An infusion pump delivers an intravenous analgesic to the patient when the patient pushes a button.
PCA reduces the wait-time to receive pain medication, reduces nursing work, and reduces the chance of medication errors.
PCA is useful for patients who have good mentation and are physically able to push the button.
1
A clinician is trying to reposition the endotracheal tube and is having some difficulty doing so; he asks for an instrument to aid in this task. Which of the following instruments is likely to be most helpfulin this scenario?
Bayonet forceps
Transfer forceps
Bozeman forceps
Magill forceps
The correct answer is:
- Magill forceps
Endotracheal intubation is an essential skill performed by multiple medical specialists to secure a patient’s airway and provide oxygenation and ventilation.
Magill forceps are angled forceps.
It guides a tracheal tube into the larynx or can also guide the nasogastric tube into the esophagus under direct vision.
These forceps can also be used to remove foreign bodies.
A 45-year-old female patient had to undergo a total abdominal hysterectomy because of menorrhagia non-responsive to pharmacological therapy. Her vital signs are stable, and the preoperative assessment revealed her to be an ideal candidate for general anesthesia. What would be the optimum time for the clinician to administer preoperative antibiotics to this patient so that she has proper antibiotic cover during the operation and does not develop a postoperative infection?
12 hours prior to the incision
60 minutes after the incision
Within 30 to 60 minutes prior to incision
Immediately after the completion of the surgery
The correct answer is:
Within 30 to 60 minutes prior to incision
The first dose is recommended to be timed to occur within 30 to 60 minutes before the surgical incision is made. If a fluoroquinolone or vancomycin is chosen for prophylaxis, the first dose should be administered within 120 minutes of the start of surgery.
The literature supports at least 30 minutes, but no greater than 60 minutes before the skin incision is made as to the optimal timing for the preoperative administration of most commonly used antibiotics. Special consideration is given for ideal preoperative timing when using a tourniquet, as the administration is least effective when the antibiotic is given after the application of a tourniquet.
Weight-based dosing should be followed per standardized protocol, and administration should occur within 1 hour of skin incision and continue for 24 hours postoperatively. Furthermore, surgical durations of greater than 4 hours or estimated blood loss over 1,500 mL necessitates repeat intraoperative dosing of antibiotics. Weight-based guidelines include A. cefazolin 2 g (3 g for weight >120 kg) as a standard adult surgical prophylaxis guideline and B. vancomycin 15 mg/kg.
Unless there is a known infection, prophylactic antibiotics should be discontinued within 24 hours. There remains controversy regarding the duration of therapy to 48 hours postoperatively following cardiothoracic surgery.
In a post-open-heart surgery patient, excessive morphine has been administered. What is the most sensitive indicator of opioid-induced respiratory depression?
- Respiratory rate
- Level of consciousness
- Confusion
- Arterial CO2
The correct answer is:
- Level of consciousness
Drowsiness after opioid is often a marker for respiratory depression.
Before the respiratory rate slows down, most people with opioid toxicity develop drowsiness or a decreased level of consciousness.
Several drugs have been developed which can block respiratory depression completely without affecting analgesia.
Once drowsiness occurs, apnea quickly follows. Naloxone can reverse this stage immediately.
Which of the following is the best method to determine if a patient has a patent airway in an emergency?
- Physical examination of the airway
- Ask the patient to speak
- The jaw thrust maneuver
- The head tilt-chin lift maneuver
The correct answer is:
- Ask the patient to speak
The best method to determine if a patient has a Peyton airway is to ask the patient to speak .
Clear speech indicates a patent airway .
Abnormal speech or the inability to speak, may indicate an obstructed airway .
The jaw thrust and head tilt-chin lift maneuvers are used to open the airway while using bag-valve-mask ventilation .
A 40-year-old patient is scheduled for laparoscopic cholecystectomy. She states that her aunt had a severe reaction to anesthesia and was in the ICU for 1 week. What is the neuromuscular-blocking agent associated with the underlying cause?
- Succinylcholine
- Vecuronium
- Rocuronium
- Atracurium
The correct answer is:
- Succinylcholine
Succinylcholine, along with halogenated inhaled volatile anesthetics, can cause malignant hyperthermia.
Malignant hyperthermia is associated with a hypermetabolic state. It is also associated with hypercarbia, acidosis, fever, circulatory collapse, and muscle rigidity.
Malignant hyperthermia is an autosomal dominant disorder that, if untreated, may cause death.
Dantrolene is the gold standard treatment for malignant hyperthermia in a dose of 2.5 mg/kg that may be repeated to a maximum of 10 mg/kg. Rarely, if symptoms persist, higher doses may be required.
A 27-year-old gravida 2 para 1 presented to the hospital in labor 6 hours ago. She is at 38 weeks gestation. She initially did not want any medication’s for analgesia, but she is now requesting an epidural for her labor pain. The on-call anesthetist explains to her that the epidural catheter will inject her epidural space with medications and will be connected to patient-controlled analgesia (PCA) pump. She is initially uncomfortable with this care plan because she is afraid of overdosing. Which of the following is the most appropriate response for her concern?
- The medications used for PCA do not have any severe or fatal side effects, so the use of more of these medications is not a concern for patient safety.
- A maximum dosage per every 24-hours will be programmed into the machine so it will be impossible for her to overdose.
- PCA has been found to reduce the length of a hospital stay when compared to traditional nurse administered scheduled dosing.
- PCA has been found to have no significant difference between opioid-related side effects compared to traditional nurse-administered schedule dosing.
The correct answer is:
- PCA has been found to have no significant difference between opioid-related side effects compared to traditional nurse-administered schedule dosing.
When PCA was compared to traditional dosing, it was documented to result and higher opioid consumption. However, those same studies also found no significant difference between opioid-related side effects, potentially indicating that the amount of opioid consumption is irrelevant.
The goal of PCA is to provide the patient with adequate pain, relief and minimize undesired outcomes. While PCA therapy has been found to consume more opioid medication, it does not increase opioid related side effects. This indicates that the goal of PCA is maintained within that data.
PCA allows patients to be in more control over their pain and helps them shift toward a more internal locus of control over their care. This explains why it has been found to increase patient satisfaction when it comes to pain control. The increased consumption of opioids without an increased side effects is worth it for the increased patient satisfaction.
Opioid medication’s can have severe and fatal side effects; extreme care needs to be taken when these medication’s are utilized. Patient CA has been found to be less cost-effective than traditional dosing and has no effect on the average length of hospital stay. PCA can be an effective way to administer analgesia and improve patient and nurse satisfaction. However, a multimodal cost-benefit analysis of the treatment plan should be performed before PCA is initiated to avoid undesired outcomes.
A 15-year old male presents with a full-thickness laceration to the right first digit. The provider effectively sutures the laceration closed. However, several hours later, the area surrounding the laceration becomes painful, dusky, and later necrotic. Which of the following is the most likely cause for this event?
- He is on calcium channel blocker for a tachyarrhythmia
- Bupivacaine
- Bupivacaine mixed with epinephrine
- Lidocaine
The correct answer is:
- Bupivacaine mixed with epinephrine
Epinephrine causes vasoconstriction. If used in the digits, nose or penis, this causes vasoconstriction that can lead to ischemia and necrosis.
Bupivacaine or lidocaine without epinephrine is indicated for local anesthetic in these circumstances.
Accidental firing of Epinephrine-pens into the thumb is a frequent cause of similar events.
Calcium channel blockers would cause vasodilation.
What type of electrolyte abnormality is expected in patients who have had a transurethral resection of the prostrate?
- Hypocalcemia
- Hypokalemia
- Hypernatremia
- Hyponatremia
The correct answer is:
- Hyponatremia
Significant absorption of non-electrolyte irrigation solutions such of as glycine, mannitol or sorbitol causes hyponatremia.
Symptoms are usually seen with drops in serum sodium exceeding 10 mEq/L.
Confusion, diminished reflexes, convulsions, stupor, or coma may occur.
The serum osmolal gap can be used to estimate the concentration of the absorbed irritant and its non-ionized metabolites.
A 42 year-old male patient is brought to the emergency department after having been thrown off his motorbike after which his lower extremities were crushed by a passing semi-truck. His vital signs are stable although the patient is unconscious and has significant crush injuries to his bilateral lower extremities below the knees. It is decided to intubate the patient to protect his airway. Which is contraindicated in this scenario?
- Cisatracurium
- Succinylcholine
- Atracurium
- Rocuronium
The correct answer is:
- Succinylcholine
Cisatracurium is a nondepolarizing paralytic and may be used in this patient. Cisatracurium causes histamine release which can cause hypotension, flushing, and broncospasm. In this patient, the depolararizing agent succinylcholine should be avoided since it may cause dangerously high levels of potassium and patients with crush injuries.
Succinylcholine should be avoided and patients with massive tissue damage, such as burns or after crush injuries. As a depolarizing agent, it can cause massive potassium effluent in these conditions leading to arrhythmias and even cardiac arrest.
Atracurium is a nondepolarizing paralytic and may be used in this patient. Atracurium is also associated with histamine release and can cause hypotension, flushing, and bronchospasm. In this patient, the depolarizing agent succinylcholine should be avoided since it may cause dangerously high levels of potassium in patients with crush injuries.
Rocuronium is a nondepolarizing paralytic and may be used in this patient. Rocuronium is metabolized by the liver, and there is a relative contraindication to this drug in patients with liver disease. In this patient, the depolarizing agent succinylcholine should be avoided since it may cause dangerously high levels of potassium in patients with crush injuries.
A 75 year-old female with a past medical history of chronic obstructive, pulmonary disease and hypertension is undergoing a femoral-popliteal bypass. After completion of the bypass, the vascular surgeon ask to reverse the heparin with 70 milligrams (mg) of protamine. The 70 mg of protamine is mistakenly injected as a single intravenous push dose. Which of the following adverse effects is most likely to have occurred as they result?
- Hypertension
- Hypotension
- Fever
- Muscle rigidity
The correct answer is:
- Hypotension
Protamine reverses the anticoagulant effects of heparin .
Protamine is derived from fish sperm and is high in arginine content.
The positively charged protein, particles and protein interact with the negatively charged heparin.
Adverse effects of protamine, including flushing, broncoconstruction, hypersensitivity, and hypotension. It is essential to give protamine a slow infusion because the hypotension can be severe.
A 17 year-old male is brought to the hospital after a fall. He is being treated with chemotherapy for leukemia. His vital signs are within normal limits. His physical exam shows a 0.5 cm linear laceration on the dorsal aspect of his left arm, which is repaired with 2-octyl cyanoacrylate. Which of the following best describes the homecare instructions for this patient?
- Apply over-the-counter topical antibiotic ointment to the area twice a day.
- Gently wash away an any crusted blood from the area.
- Allow extended bathing.
- Return to the emergency department immediately if there is redness, swelling, or drainage from the area.
The correct answer is:
- Return to the emergency department immediately if there is redness, swelling, or drainage from the area.
Two octyl cyanoacrylate has inherent antibiotic properties, and topical antibiotics may even damage the integrity of 2 octyl cyanoacrylate.
While reaching maximum bonding strength quickly , friction, such as scrubbing, can damage the 2-octyl cyanoacrylate and is not recommended.
Showering or nons sustained water exposure is not a contraindication for 2-octyl cyanoacrylate; however, prolonged exposure, such as swimming is not recommended.
While there have been studies that show no difference in infection rates between suture closure, and 2-octyl cyanoacrylate, signs of infection, such as redness, swelling, or pustular drainage should prompt presentation to medical attention.
A patient with a chest tube has bubbling in the water seal drainage during expiration. What does this indicate?
- Pneumothorax
- Emphysema.
- Lung expansion
- Air leak
The correct answer is:
- Air leak
Bubbling in the water seal chamber during expiration indicates that an air leak exists. The larger the leak the more bubbling that will occur.
A water seal chest tube drainage system will allow for air to leave the pleural space on exhalation and prevent air from entering the pleural cavity or mediastinum on exhalation. Sterile water is normally instilled up to the 2 cm line in a water seal chamber. The chest tube drainage system must be kept upright at all times and the water seal chamber monitored for evaporation of the fluid in the water seal chamber.
Valves are built into most systems to allow for high negative pressure and maintaining the water seal. High negative pressure situations can include patients with vigorous coughing, respiratory distress, milking of the tube, aggressively or disconnected/decreased suction.
Normally wall suction is set on 80 to 100 mmHg to provide a gentle bubbling in the suction control chamber. Otherwise, aggressive bubbling causes evaporation of the water in the suction control chamber, and ultimately decreases the suction pressure. Dry suction drainage systems have no bubbling and no evaporation of water and decrease suction risks. A regulator is used to control the amount of suction by turning a knob or dial to the appropriate level. A red stripe will appear in the window near the prescribed suction level. The dry drainage system comes preset at -20 cm for ease of use and can be adjusted. The wall section will usually need to be set at 80 to 100 mmHg. An orange float will appear in the indicator window. Enough suction must be applied for the float to work. Drainage systems have built-in positive pressure valves as well to protect against positive pressure complications. If a chest tube is clamped, a tension pneumothorax can result.
A 65 year-old woman is undergoing a two level decompression and fusion of the lumbar spine. Neostigmine was administered for the reversal of a non-depolarizing blocking agent. Which of the following describes the potential side effects of the administration of the cholinergic drug?
- Bradycardia
- Decreased secretions
- Mydriasis
- Tachycardia
The correct answer is:
- Bradycardia
One of the cardiac muscarinic effects of neostigmine is bradycardia.
Neostigmine causes cholinergic side effects, including increased secretions.
A cholinergic side effect of Neostigmine is miosis.
Neostigmine causes bradycardia as a resource cholinergic stimulation, not tachycardia.
A 12 year-old obese male with a past medical history of sleep apnea, recurrent strip pharyngitis and recurrent otitis media presents in the pre operative bay as he is scheduled to undergo a tonsillectomy with adenoidectomy. Which of the following is a potential complication of inadequate, postoperative pain control associated with his procedure?
- Dehydration
- Fatigue
- Hoarseness
- Dyspnea
The correct answer is:
- Dehydration
Inadequate postoperative pain can manifest with a variety of adverse somatic symptoms. Symptoms of hypertension, diaphoresis, nausea, vomiting, tachycardia, and anxiety are common among most cases of inadequate pain control in the particular case of tonsillectomy dehydration is a common adverse effect that occurs from inadequate pain relief during swallowing .
When assessing for adequate pain control it is not sufficient to evaluate the patient’s pain level only at rest . A patient’s pain level with activity is an important thing to consider, as well. Uncontrolled pain with activity may impair a patient’s ability to perform activities of daily living or participate in rehab exercises.
Questions to ask during a thorough postoperative pain assessment should aim to determine pain onset, location, quality of the pain, intensity, alleviating and aggravating factors, and the effect of current treatment modalities being used. Patients should also be assessed for any factors that may influence their ability to reliably report their level of pain, such as with the presence of cognitive or communication barriers.
A validated pain assessment tool such as the visual analog scale or faces pain scale should be used to track postoperative pain, treatment effectiveness .
A 58 year- old man is about to undergo elective surgery. He works in a bank and has no significant past medical history. During the preoperative assessment before the procedure, the healthcare provider notes that the heart rate is 45/min. Which of the following is the most appropriate initial step in the management of this client?
- Check troponin levels.
- Give atropine.
- Postpone the surgery.
- Insert a temporary intravenous pacemaker.
The correct answer is:
- Postpone the surgery
Sinus bradycardia is a cardiac rhythm with appropriate cardiac muscular depolarization initiating from the sinus node generating less than 60/min.
If a client is found to have a low heart rate during an elective surgical procedure, the surgery should be canceled .
Sinus bradycardia, like any of the other bradyarrhythmias, is caused by a multitude of intrinsic and extrinsic factors that may compromise the intergovernmental the sinus node. These factors can cause the failure of the impulse formation at the sinus node, impulse conduction at the atrioventricular node, or bundle of His-Purkinje fibers.
The best step would be to postpone the procedure and refer him to a cardiologist for further up .
A 63 year-old is about to undergo an elective laparoscopic cholecystectomy. In the preoperative area, it is determined that the patient’s blood pressure is high because she did not know that she could take her blood pressure medications with sips of water. The anesthesiologist agrees to go ahead with the surgery. What anesthetic will he preferably avoid in this patient?
- Halothane
- Enflurane
- Ketamine
- Succinylcholine
The correct answer is:
- Ketamine
Ketamine is an agent for induction and maintenance of Anesthesia .
It is a good analgesic agent that is also used in the intensive care unit .
During induction, the drug can cause an elevation and blood pressure and muscle tremors .
It is a useful agent when inducing patients who have low blood pressure.
A 57 year-old male is brought to the emergency department with a burn injury over more than 50% of the body surface area. The client is developing signs of respiratory failure the providers plan to intubate the client. What neuromuscular agent is contraindicated in the client?
- Rocuronium
- Atracurium
- Norcurium
- Succinylcholine
The correct answer is:
- Succinylcholine
Succinylcholine is used for rapid sequence intubation, as is very short-acting. It is a depolarizing, skeletal muscle relaxant. It bonds with motor endplate cholinergic receptors producing depolarization.
The neuromuscular block causes flaccid paralysis after initial fasciculations. Succinylcholine can worsen the effects of hyperkalemia.
It must be used with caution and clients with burns traumatic wounds, crush injuries, or hyperkalemia. It also may cause problems with those receiving dioxin or quinidine, arrhythmias or cardiac arrest are possible.
Caution should be employed when using it on clients with renal, pulmonary, cardiovascular, hepatic or cardiac disease.
A patient is brought to the emergency department with shortness of breath. The attending clinician notes that the oxygen saturation of the patient is dropping along with the blood pressure. The attending clinician decides to secure the airway by intubating the patient. He believes the patient may require an endotracheal tube for an extended period of time and may become obstructed. What built-in anatomical mechanism of the endotracheal tube is meant to overcome distal obstruction?
- Bevel
- Murphy’s eye
- Distal balloon
- Distal airway safety mechanism
The correct answer is:
- Murphy’s eye
The Murphy’s eye is a safety mechanism built into the endotracheal tube. If the distal end of the endotracheal tube should become obstructed by the wall of the trachea, gas flow can still occur via Murphy’s eye, positioned in the lateral wall of the tube. This prevents complete obstruction of the tube.
The bevel is meant to facilitate placement into the trachea without injuring the vocal cords.
A cuff is an inflatable balloon at the distal end of the endotracheal tube. The inflated cuff forms a seal against the tracheal wall. This prevents gastric contents from entering the trachea and facilitates the execution of positive pressure ventilation, however, it does not prevent obstruction of the tube.
The “distal airway safety mechanism” is a fictional name for a portion of the tube than does not exist.
A 65 year-old man is undergoing surgery for a right knee replacement, and he is receiving isoflurane general anesthesia. The anesthesiologist notices excessive saliva formation during the surgery. Which of the following medications is most appropriate for this side effect in this patient?
- Diphenhydramine
- Atropine
- Dexamethasone
- Neostigmine
The correct answer is:
- Atropine
Atropine is widely used with anesthesia to decrease oral secretions.
Atropine is administered via intravenous injections and works immediately.
Atropine increases heart rate, dilates pup, causes dry skin, and anesthetizes the nerve endings in the skin.
Atropine also relaxes smooth muscle and suppresses gland and mucous secretions. In the past, it was used to treat peptic ulcers by reducing the production of stomach acid.
A 25-year-old intoxicated man is brought to the emergency department after a knife fight. He has no significant past medical history. He has multiple lacerations to his face, including his eyebrow, forehead, peri-auricular region, and jaw. Bilateral supraorbital and supratrochlear nerve blocks are performed, and multiple other facial lacerations are repaired after local infiltration with bupivacaine. The patient is getting prepared for discharge when he suffers a seizure. He is given intramuscular lorazepam, and the seizure activity abates. Which of the following is the next best step in the management of this patient?
- IV lipid emulsion
- IV lorazepam infusion
- IV hypertonic saline
- IV vitamin B6
The correct answer is:
- IV lipid emulsion
This patient had a total of four nerve blocks as well as a large amount of local infiltration. He is likely suffering from local anesthetic systemic toxicity.
The treatment of local anesthetic systemic toxicity is intravenous lipid emulsion.
Local anesthetic systemic toxicity symptoms can include central nervous system changes, arrhythmias, seizures, respiratory arrest, and coma.
Benzodiazepine can increase seizure threshold but does not affect the other actions of local anesthetic systemic toxicity. Hypertonic saline is used for seizures caused by hyponatremia. Vitamin B6 is used for seizures caused by isoniazid toxicity. This patient does not have any medical history, and thus the most likely cause of his seizure is local anesthetic systemic toxicity.
A 65-year-old male presents to the emergency department with acute onset generalized abdominal pain, nausea, and vomiting. On examination, there is marked abdominal rigidity and guarding. No bowel sounds are audible. After a detailed evaluation, he was diagnosed with intestinal perforation. He is being prepped for surgery, and the nurse administers an intravenous antibiotic before the procedure. What is the role of administering this antibiotic?
- Treat underlinfection
- Maintain the serum antibiotic level until the surgery is over
- Decrease the risk of infection after surgery
- Protect the patient in the event of an aseptic lapse
The correct answer is:
- Decrease the risk of infection after surgery
Preoperative antibiotic prophylaxis is defined as administering antibiotics prior to performing surgery to help decrease the risk of postoperative infections.
The routine administration of prophylactic antibiotics is standard in surgeries in which extensive dissections and higher amounts of anticipated blood loss are expected.
In most surgeries, the intent is to ensure the bactericidal concentration has been reached in the blood and tissues before incision. In this case, the patient already has bacterial contamination of the peritoneum due to the perforation.
Cephalosporins are most commonly used. Other commonly used antibiotics include vancomycin, metronidazole, aminoglycosides, and clindamycin.
A 65-year-old male presents to the emergency department with acute onset generalized abdominal pain, nausea, and vomiting. On examination, there is marked abdominal rigidity and guarding. No bowel sounds are audible. After a detailed evaluation, he was diagnosed with intestinal perforation. He is being prepped for surgery, and the nurse administers an intravenous antibiotic before the procedure. What is the role of administering this antibiotic?
- Treat underlinfection
- Maintain the serum antibiotic level until the surgery is over
- Decrease the risk of infection after surgery
- Protect the patient in the event of an aseptic lapse
The correct answer is:
- Decrease the risk of infection after surgery
Preoperative antibiotic prophylaxis is defined as administering antibiotics prior to performing surgery to help decrease the risk of postoperative infections.
The routine administration of prophylactic antibiotics is standard in surgeries in which extensive dissections and higher amounts of anticipated blood loss are expected.
In most surgeries, the intent is to ensure the bactericidal concentration has been reached in the blood and tissues before incision. In this case, the patient already has bacterial contamination of the peritoneum due to the perforation.
Cephalosporins are most commonly used. Other commonly used antibiotics include vancomycin, metronidazole, aminoglycosides, and clindamycin.
A 75-year-old American Association Anesthesiology class-III (ASA-III) man with a history of coronary artery disease, chronic renal disease, and obstructive sleep apnea is taken to the operating room for inguinal hernia repair. He had been seen by a cardiologist and cleared for surgery. General anesthesia via laryngeal mask airway (LMA) is considered with maintenance using desflurane. Vital signs are normal prior to pre-op, and induction and placement of the LMA are uneventful. Ten minutes later, during surgical prep, the patient’s vital signs show HR 115/min, BP 148/96 mmHg, RR 14/min, SpO2 99%, EtCO2 45 mmHg, and T 98.9 F (37.1 C). The end-tidal desflurane value reads 8.5, and the only other medication administered at this point was cefazolin. What is the most likely cause for the patient’s clinical condition?
- Rapid titration of desflurane
- Malignant hyperthermia
- Insufficient depth of anesthetic
- Allergic reaction to perioperative antibiotic administered
The correct answer is:
- Rapid titration of desflurane
Desflurane, especially when rapidly titrated, will stimulate the sympathetic nervous system.
A higher flow rate on the anesthesia machine can lead to an increase in the fraction of inspired volatile, which will yield more rapid titration of anesthetic. This rapid change in end-tidal volatile, rather than a slow titration, can increase HR and BP transiently.
In this patient with a history of CAD, sympathetic stimulation should be avoided to prevent further ischemia.
While desflurane is a malignant hyperthermia triggering agent, it is unlikely given the patient’s modest elevation in EtCO2, which is more likely explained by his history of OSA. The elevation in HR and BP are non-specific, and it would be much more likely to malignant hyperthermia with a much more rapid elevation in EtCO2. The non-specific nature of isolated tachycardia and hypertension are much more commonly associated with rapid titration of desflurane as opposed to MH in this scenario. Inadequate depth of anesthesia certainly presents as hypertension and tachycardia. However, it is much less likely in this scenario where the patient has no surgical stimulation yet as the prep is ongoing, and it has been 10 minutes since the stimulation of placing the LMA occurred. In addition, the patient’s MAC is > 1, which is more than adequate given the lack of stimulation at this time. It is because of the rapid titration of the volatile agent desflurane that likely led to tachycardia and hypertension. Allergic or anaphylactic reactions are common with steroidal non-depolarizing neuromuscular blocking drugs as well as perioperative antibiotics. However, it is much more commonly associated with hypotension, tachycardia, and increased peak airway pressures. The etiology of this patient’s tachycardia and hypertension are much more likely caused by sympathetic stimulation from desflurane. The mechanism by which the sympathetic stimulation occurs is not fully understood, but rapid titration tends to increase the incidence of stimulation.
A 51-year-old man is scheduled for a laparoscopic cholecystectomy under general anesthesia. His past medical history is significant for hypertension and atrial fibrillation. His current medications include lisinopril, atorvastatin, and warfarin. He is taking warfarin 2.5 mg on alternate days. During the preoperative anesthetic assessment, lab results show an INR of 2.6. Which of the following is the most appropriate plan for proceeding with the surgery?
- Proceed with surgery under general anesthesia
- Proceed with surgery using spinal anesthesia
- Stop warfarin 5 days before surgery and start bridging therapy with enoxaparin
- Give low-dose vitamin K and proceed with surgery
The correct answer is:
- Stop warfarin 5 days before surgery and start bridging therapy with enoxaparin
To proceed with the surgery, warfarin must be stopped 5 days before the surgery.
Bridging therapy with enoxaparin should be started 3 days before the surgery, and the patient’s INR should be retested.
Low-dose vitamin K reversal can help patients with an INR of 1.5 to 1.9 (phytomenadione 1 to 3 mg IV). This, however, will take a few hours to have an effect. Patients should have their warfarin reintroduced as soon as possible following surgery, according to their pre-procedure protocol.
It is imperative that enoxaparin be continued post-operatively, together with warfarin, until the INR reaches therapeutic levels.
An operating room nurse observes a red rash under the elastic bandage on a patient’s wrist and around his IV insertion site during a perioperative assessment. The patient reports that the area itches intensely. The nurse also notices that the patient is wheezing. What is the preferred next step in management?
- Emergent complete blood count with differential should be requested
- An intravenous anti-histamine should be started immediately
- Intravenous methylprednisolone should be started immediately
- The surgeon and anesthetist should be notified immediately
The correct answer is:
- The surgeon and anesthetist should be notified immediately
Elastic bandages and IV tubing contain latex to make them stretchy.
Symptoms of latex allergy include redness, rash, and asthmatic reaction on contact.
Latex allergy, however, can be mistaken for an irritant or allergic contact dermatitis.
If latex allergy is suspected or confirmed, the best next step is to notify the surgeon and anesthetist immediately.
A 65-year-old male with metastatic tongue carcinoma is undergoing partial glossectomy with neck dissection. The patient is administered an intravenous anesthetic followed by successful nasal intubation. The surgeon initially notices difficulty opening the mouth due to masseter spasm, but proceeds with the surgery. Minutes later, the anesthesia monitor alarms due to an increase in end-tidal carbon dioxide. The patient’s heart rate is 120/min, and unresponsive to opioid medication. The temperature is 98.7 F. Which of the following is the most common intravenously administered agent associated with the patient’s current condition?
- Sevoflurane
- Succinylcholine
- Etomidate
- Ketamine
The correct answer is:
- Succinylcholine
Succinylcholine is a noncompetitive neuromuscular blocking drug, which is the most common cause of malignant hyperthermia even when including inhaled volatile agents.
The patient in the scenario shows early signs of malignant hyperthermia, including masseter muscle contraction and unexplained tachycardia.
Late signs include sustained muscle rigidity, metabolic acidosis, myoglobinuria, and increasing temperature.
Ketamine is similar in structure to phencyclidine (PCP) and acts as a dissociative anesthetic. It produces feelings of detachment from self and environment. This drug tends to cause copious secretions and respiratory depression and should be used cautiously in patients with chronic obstructive pulmonary disease and asthma. Additional side effects include hallucinations and laryngospasm.
A 32-year-old woman, gravida three para two, presents with the complaint of inability to pass urine or flatulence 8 hours after surgery. She had an emergency laparoscopic left salpingectomy for a ruptured ectopic pregnancy. She was transfused with four units of packed red blood cells and received 3 liters of normal saline during the surgery. A foley catheter was placed before the surgery and was removed after the surgery. The intraoperative urine output was 250 milliliters. The operation was successful and had no complications. Since then, the patient has not been able to pass urine or flatulence and is feeling nauseous. She has no fever, vomiting, or abdominal pain. Her blood pressure is 129/89 mmHg, and her pulse is 81 beats per min. On examination, the abdomen is soft but distended. Suprapubic tenderness is present. There is no rebound or guarding. The surgical incisions are clean and dry. Which of the following is the cause of this patient’s condition?
- Urinary retention
- Ureteral injury
- Large bowel obstruction
- Intraoperative bladder injury
The correct answer is:
- Urinary retention
Patients who are unable to void after a surgical procedure would likely have postoperative urinary retention. The incidence of postoperative urinary retention is about 4 to 13 percent overall. The rate of postoperative urinary retention after a cesarean section done with epidural anesthesia is roughly 23 to 28 percent.
Risk factors for postoperative urinary retention include age over 50, surgery for more than 2 hours, receiving more than 750 milliliters of fluids during the surgical operation, regional anesthesia, neurological disease, previous pelvic surgery, or underlying bladder dysfunction.
In this patient, the combination of anesthesia and intraoperative fluids increased her risk for postoperative urinary retention. The anesthesia causes decreased detrusor muscle contractility, and large volumes of intraoperative fluids cause bladder overdistension.
Patients present with an inability to void, suprapubic tenderness, and abdominal distension. Placement of a urinary catheter can simultaneously diagnose and treat the condition. The catheter allows the urine to flow out of the bladder, thus decreasing the overdistension injury of the bladder.
Shortly after an appendectomy under general anesthesia, a patient develops tachycardia, tachypnea, fever, and muscle rigidity. A diagnosis of malignant hyperthermia is made, and dantrolene sodium is administered. Which of the following best describes the mechanism of action of this drug?
- Decreases the release of calcium from the sarcoplasmic reticulum by binding to the ryanodine receptor
- Increases the levels of potassium by depolarization of the neuromuscular junction
- Inhibits the release of dopamine
- Blocks neuromuscular transmission
The correct answer is:
- Decreases the release of calcium from the sarcoplasmic reticulum by binding to the ryanodine receptor
Dantrolene sodium is the only approved treatment for malignant hyperthermia.
Dantrolene binds to the ryanodine receptor and inhibits calcium release into the sarcoplasmic reticulum, which prevents sustained muscle contraction.
Besides dantrolene, dopamine agonists can be helpful for malignant hyperthermia. Dantrolene does not increase the levels of potassium ions. However, the use of dantrolene, along with calcium channel blockers, can result in hyperkalemia and can cause cardiac depression. Therefore, the use of calcium channel blockers with dantrolene is contraindicated.
Dantrolene does not block neuromuscular transmission and does not interfere with the reversal of muscle relaxants.
A 55-year-old woman with a body mass index of 37 kg/m2 presents to the preoperative clinic before a right breast lumpectomy. Her blood pressure is 145/89 mmHg in the right arm and 146/90 mmHg in the left arm. Repeat measurements 20 minutes later are similar. She drinks a glass of wine two to three nights weekly. She does not smoke and denies illicit drug use. When asked if she has a history of hypertension, she states her clinician diagnosed white-coat hypertension three months ago, based on ambulatory blood pressure monitoring. The patient said she was supposed to wear the monitor for one day but, since the clinic was closed on a holiday, she kept the monitor on for two days. The patient states her blood pressure on ambulatory blood pressure monitoring averaged 127/78 mmHg. What is the best next step in the management of this patient?
- Repeat ambulatory blood pressure monitoring for 72 hours and return to review the results
- Initiate amlodipine and follow up in one week
- Initiate lisinopril and follow up in one week
- Confirm white-coat hypertension with her outside provider then proceed with the surgery
The correct answer is:
- Confirm white-coat hypertension with her outside provider then proceed with the surgery
White-coat hypertension is best diagnosed by ambulatory or home blood pressure monitoring. It is usually performed for 24 hours and takes multiple readings. A reliable assessment should include at least 70% of expected readings and at least 20 awake and seven asleep blood pressure measurements. This patient’s ambulatory monitor was on for 48 hours, which is more than sufficient to support the diagnosis of white-coat hypertension.
White-coat hypertension has been associated with slightly increased cardiovascular disease risk and all-cause mortality risk.
One to five percent of patients with white-coat hypertension convert to sustained hypertension annually. The incidence of conversion to sustained hypertension is greater with older age, obesity, higher initial blood pressure readings, and in patients of African American descent.
Correct identification of white-coat hypertension is needed to avoid unnecessary prescription and expenditure on antihypertensive drugs.
A 35-year-old female is undergoing emergency appendectomy under general anesthesia. Induction was achieved via rapid sequence intubation using rocuronium as the muscle relaxant. She has a BMI of 30 kg/m2. Past medical history is insignificant. The intraoperative course is uneventful. After the procedure, the patient is given a medication for the reversal of the effects of neuromuscular blockade. The patient develops bradycardia. The vital signs are heart rate 42/min, blood pressure 90/60 mmHg, respiratory rate 15/min, and temperature 98.6 F (37 °C). What is the next step in the management of this patient?
- Administer adrenaline
- Administer ephedrine
- Administer anticholinergic
- Give intravenous fluids
The correct answer is:
- Administer anti cholinergic
Sugammadex is an agent for the reversal of neuromuscular blockade induced by rocuronium and vecuronium in general anesthesia. It is the first selective relaxant binding agent.
Bradycardia has been noted for patients, with some leading to cardiac arrest. Anticholinergics can be administered to counteract marked bradycardia.
A relatively common complication is hypotension and bradycardia; hence resuscitative equipment and anticholinergics must be available in the room before administration of this drug. Sugammadex is otherwise relatively safe as long as it is utilized in a monitored setting like the operating room and ICU.
It is repeatedly stated that the incidence of bradycardia after the administration of sugammadex is in the order of 1%. It is suggested that, although the bradycardia may lead to cardiac arrest, it is only a transient effect and is dose-related.
A 16-year-old trauma patient has an epidural infusion of morphine before his knee surgery. Three hours later, he reports itching, and his respiratory rate is four breaths per minute upon assessment. He opens his eyes when you speak to him for a couple of seconds and then immediately falls back asleep during your interview. Which of the following medications is the most appropriate treatment for this patient?
- Naloxone
- Oxygen
- Diphenhydramine
- Intravenous fluids
The correct answer is:
- Naloxone
Naloxone is an opioid antagonist often used to counter the respiratory depression following morphine overdose.
Naloxone has an affinity for the mu-receptors and rapidly reverses the effects of morphine.
Naloxone is commonly administered intravenously but can also be given via intramuscular or intranasal routes.
A small initial dose, usually 0.04 to 0.1mg IV is recommended in opioid-dependent patients with symptoms of opioid overdose to avoid opioid withdrawal symptoms. Larger initial dosing, such as 0.4mg IV, is recommended in patients without a known history of opioid dependency.
A 63-year-old female patient is undergoing a laparoscopic appendectomy. She has no relevant past medical history and weighs 68 kilograms. She takes a daily multivitamin and exercises regularly. She denies any alcohol, tobacco, or illicit drug use. She was induced with propofol, given 40 milligrams of rocuronium, and 180 milligrams of succinylcholine. She was then subsequently intubated with a 7.5-millimeter endotracheal tube with no difficulty. At the end of the case, she is found to have four twitches with the nerve stimulator. Which of the following medications should be administered along with 0.6 mg of glycopyrrolate?
- Physostigmine 2 milligrams
- Scopolamine 100 micrograms
- Neostigmine 3 milligrams
- Pyridostigmine 5 milligrams
The correct answer is:
- Neostigmine 3 milligrams
Physostigmine and pyridostigmine are anticholinesterase medications. They are most commonly used outside the intraoperative setting for conditions such as myasthenia gravis.
Neostigmine is the drug of choice for reversal of neuromuscular blockade.
Neostigmine is commonly dosed by weight and combined with glycopyrrolate.
Neostigmine is an anticholinesterase and is combined with an anticholinergic medication to help increase the amount of acetylcholine available at the neuromuscular junction for adequate reversal.
A 54-year-old male patient had abdominal surgery for gastric cancer and receives many units of transfused blood. Post-surgery he complains of numbness around his mouth and carpopedal spasm. ECG reveals a prolonged QT interval. What is the preferred next step management?
- Infusion of a couple of bicarbonate vials
- Infusion of 2 mEq/L potassium chloride
- Infusion of intravenous calcium gluconate
- Infusion of intravenous 50% dextrose
The correct answer is:
- Infusion of intravenous calcium gluconate
After a massive blood transfusion, one can develop hypocalcemia due to the excess citrate in the blood.
Each unit of packed red cells for transfusion contains about 2 to 3 mg of citrate. This concentration of citrate is normally cleared within a few minutes from the body by the liver.
However, ill and traumatized patients who require multiple units of packed red blood cells may have poor elimination of citrate. Also, the high citrate dose may persist in the body longer.
Thus, if the patient is symptomatic from hypocalcemia, one may have to administer intravenous calcium gluconate.
Which of the following is the most important management strategy to prevent central line insertion site infection?
- Daily dressing changes with the use of transparent dressing and antimicrobial-impregnated discs
- Placing internal jugular central lines with line change every 2 weeks
- Aseptic technique for transparent dressing changes every 5 to 7 days
- Aseptic technique for transparent dressing changes every 2 to 3 days
The correct answer is:
- Aseptic technique for transparent dressing changes every 5 to 7 days
Central line dressings do not need frequent changes. A gauze-based dressing may be changed every two days, and transparent dressing may be changed every 5 to 7 days. More frequent dressing changes need only occur if the dressing is visibly soiled or comes loose.
Each lumen of the central line should be assigned a distinct purpose. Frequent manipulation of the line may lead to higher catheter-based complications.
Prophylactic antibiotics have not been shown to reduce central line insertion site infections.
Aseptic technique during dressing changes ensures that no new bacteria are introduced to the insertion site. Furthermore, proper hand hygiene and the use of gloves for any line manipulation will reduce the risk of catheter infections.
A 35-year-old female presents to the surgical clinic for evaluation. She has been experiencing pain in her left big toe. Examination reveals mild redness and minimal discharge. The medial edge of the nail is seen penetrating the cuticle on the medial aspect. It is decided to cut out the affected nail. The patient’s toe is draped with povidone solution. A solution of an amino-amide agent buffered with bicarbonate is injected along the metatarsal head. This is followed by the removal of the affected nail. What physiological effect will most likely be observed after the injection of the aforementioned agent?
- Reduced conductance across sodium channels
- Increased conductance across potassium channels
- Reduced conductance across calcium channels
- Increased conductance across chloride channels
The correct answer is:
- Reduced conductance across sodium channels
This individual has presented with pain and swelling of the left big toe. Her examination reveals the presence of ingrown toenail along with perilesional edema and erythema. A minor surgical procedure is being performed to remove the affected nail. The procedure is painful and requires the administration of a local anesthetic to numb the affected area. The agent used in this case is lidocaine an amino-amide agent.
Lidocaine functions by binding to the sodium channels and reducing sodium conductance across the neuronal membrane. Reduced sodium conductance impairs the propagation of action potential across the nerve fibers and is responsible for the anesthetic action of the agent.
Binding to the transmembrane sodium channel and inhibition of influx has been postulated to be the primary mechanism of action. Since type C fibers, that carry pain and temperature, are unmyelinated these fibers are affected first.
Local anesthetic can be used safely to provide pain relief during minor surgical procedures. They usually do not affect calcium and chloride conductance. A reduction in potassium conductance may also contribute to the effect of local anesthetics.
Prior to induction of anesthesia, a patient is breathing spontaneously with oxygen provided by an anesthesia machine by gently resting a mask connected to the common gas outlet over the patient’s mouth. The patient is then induced with intravenous anesthesia and becomes apneic. The anesthesiologist wishes to provide positive pressure ventilation for the patient without turning on the ventilator (leaving the ventilator on a manual/spontaneous mode). Closing which of the following anesthesia machine components will allow for positive pressure ventilation to be performed by squeezing the reservoir bag?
- Inspiratory unidirectional valve
- Expiratory unidirectional valve
- Adjustable pressure limiting valve
- Common gas outlet
The correct answer is:
- Adjustable pressure limiting valve
The adjustable pressure limiting valve exists to relieve pressure to prevent barotrauma to the patient and damage to the flowmeters and vaporizers.
When the adjustable pressure limiting valve is open, it allows for the patient to breathe spontaneously.
Closing the adjustable pressure limiting valve to some degree will allow for ease of positive pressure ventilation once the patient becomes apneic.
Failing to close the adjustable pressure limiting valve after induction will result in difficulty ventilating the patient using the machine to bag-mask ventilate the patient.
A 7-year-old female undergoes an emergency appendectomy. The surgery is a success. However, there is difficulty managing her post-operative pain. She has been reaching all of her age-appropriate milestones and does not have any previous past medical history. She states her pain is a 3 out of 10. Today is postoperative day number one. The surgical incision site is clean and dry, and the patient’s vitals are largely within normal limits, except for a mild tachycardia attributed to her pain with movement. She is tolerating her diet but has yet to have a bowel movement. She has ambulated minimally due to her pain. What is the preferred way to assess her pain?
- Self-reporting pain scale
- Use a visual analog scale
- Numerical pain scale
- Observational pain assessment
The correct answer is:
- Use a visual analog scale
An age-based rating scale of pain should be used for this normally developed seven years old. A visual analog scale should be implemented to asses her pain.
A visual analog scale presents faces in various degrees of distress for the child to identify.
There are numerous pain assessment tools used in adults and children. However, this child is likely too young for a numerical scale. Although she reported a 3/10 in pain severity, she has ambulated minimally and has vital sign changes due to her pain. A visual scale would be the best option for her age group.
Visual analog scales do not need to be used exclusively on children. They are often used both on adults and children to help them better interpret the numerical values they assign to their pain severity.
A 65-year-old man is admitted to the medical intensive care unit for shortness of breath. A decision is made to intubate this patient. The provider instructs the nurse, “Please draw up fifteen milligrams of etomidate and fifty milligrams of rocuronium.” Which of the following is the next most appropriate course of action in this scenario?
- Proceed as instructed emergently
- Document the instruction and proceed
- Report the provider to the hospital administrator
- Stop the provider and request for an order and a timeout
The correct answer is:
- Stop the provider and request for an order and a timeout
A universal timeout detailing the procedure, equipment, medications, and plan is an effective way of reducing critical errors during a procedure.
A written order can ensure the right dose for the right patient. A written order is a must prior to proceeding with medication delivery to reduce critical errors.
Peer dose checking is also an effective way of reducing medication errors for high-risk medications like insulin, blood thinners, and paralytics.
Verbal read back nor only reduces the errors in similar name medications, it can also reduce errors in dosages. Reporting the provider is unnecessary at this point.
A 23-year-old female underwent a scheduled cesarian section for a full-term intrauterine pregnancy 12 hours ago. She received a single injection, subarachnoid block which contained fentanyl, morphine, and bupivacaine. She now is complaining of severe itching while on the mother-baby unit. Which of the following medications would be most beneficial to decrease her pruritus?
- Diphenhydramine
- Nalbuphine
- Naloxone
- Propofol
The correct answer is:
- Nalbuphine
Nalbuphine has been found to be superior in treating opioid-induced pruritus when compared to diphenhydramine, naloxone, or propofol in patients receiving neuraxial opioids for surgery or childbirth.
Nalbuphine is classified as a mixed opioid receptor agonist-antagonist.
As with all opioid medications, treatment of nalbuphine toxicity or adverse effects is with the antidote naloxone.
Nalbuphine is available as an intravenous, subcutaneous, or intramuscular injection. It is not available in oral form due to poor oral bioavailability.
A 59-year-old man is being evaluated after undergoing elective, outpatient hernia repair. He has not urinated for 8 hours. Examination reveals suprapubic tenderness and a feeling of discomfort on palpation. The patient has the urge to urinate but is unable to void. What intraoperative factor is most often implicated for this surgical complication?
- The use of electrocautery in the inguinal canal
- Intraoperative Valsalva per anesthesia to test the hernia repair
- Supine positioning of the patient for the procedure
- The total volume of intraoperative intravenous fluids given
The correct answer is:
- The total volume of intraoperative intravenous fluids given
Postoperative urinary retention (POUR) is not an uncommon problem. The incidence of reported POUR in the literature has extreme variability between 5%–70% depending upon the study cited.
Intraoperative risk factors leading to POUR include operative time, intraoperative intravenous fluid volume and type of anesthetic used.
Intraoperative intravenous fluid volume seems to lead to a higher rate of POUR when a catheter is not placed preoperatively, theoretically due to stretching of a non-drained bladder while the surgery is ongoing.
Operating time greater than 2 hours was a significant predictor of POUR in one study. Another study of total joint patients found the risk of POUR increasing 25% every 15 minutes spent in the operating room.
A 52-year-old man is to undergo a routine open inguinal hernia repair. He has no past medical history and takes no regular medications. In the anesthetic room, he is given propofol, fentanyl, and succinylcholine intravenously. Absence of which of the following enzymes could prolong the neuromuscular blockade effects induced by one of the anesthetic drugs?
- Glucose-6-phosphate dehydrogenase
- Plasma cholinesterase
- Dopamine beta-hydroxylase
- Cytochrome P450 oxidase
The correct answer is:
- Plasma cholinesterase
Succinylcholine chloride is a short-acting depolarizing neuromuscular blockade that is approved by the United States Food and Drug Administration (FDA) as a provision to other sedatives or hypnotics. It is a correlate of acetylcholine (ACh); hence, it disrupts all cholinergic receptors of the parasympathetic and sympathetic nervous systems.
Decreased levels of plasma cholinesterase are seen in patients with genetic defects, pregnancy, malignancy, severe liver disease, chronic renal failure, burns, infections, anemia, myxedema, and peptic ulcer disease.
This can lead to prolonged effects of succinylcholine, including complete skeletal muscle paralysis. Hyperkalaemia can also occur.
The activity of plasma cholinesterase can be decreased by the use of oral contraceptives, corticosteroids, MAO inhibitors, or organophosphate insecticides.
Shortly after an appendectomy under general anesthesia, a patient develops tachycardia, tachypnea, fever, and muscle rigidity. A diagnosis of malignant hyperthermia is made, and dantrolene sodium is administered. Which of the following best describes the mechanism of action of this drug?
- Decreases the release of calcium from the sarcoplasmic reticulum by binding to the ryanodine receptor
- Increases the levels of potassium by depolarization of the neuromuscular junction
- Inhibits the release of dopamine
- Blocks neuromuscular transmission
The correct answer is:
- Decreases the release of calcium from the sarcoplasmic reticulum by binding to the ryanodine receptor
Dantrolene sodium is the only approved treatment for malignant hyperthermia.
Dantrolene binds to the ryanodine receptor and inhibits calcium release into the sarcoplasmic reticulum, which prevents sustained muscle contraction.
Besides dantrolene, dopamine agonists can be helpful for malignant hyperthermia. Dantrolene does not increase the levels of potassium ions. However, the use of dantrolene, along with calcium channel blockers, can result in hyperkalemia and can cause cardiac depression. Therefore, the use of calcium channel blockers with dantrolene is contraindicated.
Dantrolene does not block neuromuscular transmission and does not interfere with the reversal of muscle relaxants.
A 25-year-old intoxicated man is brought to the emergency department after a knife fight. He has no significant past medical history. He has multiple lacerations to his face, including his eyebrow, forehead, peri-auricular region, and jaw. Bilateral supraorbital and supratrochlear nerve blocks are performed, and multiple other facial lacerations are repaired after local infiltration with bupivacaine. The patient is getting prepared for discharge when he suffers a seizure. He is given intramuscular lorazepam, and the seizure activity abates. Which of the following is the next best step in the management of this patient?
- IV lipid emulsion
- IV lorazepam infusion
- IV hypertonic saline
- IV vitamin B6
The correct answer is:
- IV lipid emulsion
This patient had a total of four nerve blocks as well as a large amount of local infiltration. He is likely suffering from local anesthetic systemic toxicity.
The treatment of local anesthetic systemic toxicity is intravenous lipid emulsion.
Local anesthetic systemic toxicity symptoms can include central nervous system changes, arrhythmias, seizures, respiratory arrest, and coma.
Benzodiazepine can increase seizure threshold but does not affect the other actions of local anesthetic systemic toxicity. Hypertonic saline is used for seizures caused by hyponatremia. Vitamin B6 is used for seizures caused by isoniazid toxicity. This patient does not have any medical history, and thus the most likely cause of his seizure is local anesthetic systemic toxicity.
A 70-year-old man with a history of end-stage renal disease and postoperative nausea and vomiting undergoes an elective inguinal hernia repair. He was started on transdermal scopolamine the night before surgery. His vital signs preoperatively include a pulse of 88/min and a temperature of 36.1 C (97 F). He is administered postoperative nausea and vomiting (PONV) prophylaxis with several medications. His intraoperative course was uneventful. In PACU, he is flushed, becomes more confused compared to baseline, and complains of a dry mouth. His heart rate increases to 132/min, and his temperature increases to 99.8 F. What is the next best step in the management of this patient?
- Apply ice-cooling blankets
- Remove the scopolamine patch placed preoperatively
- Administer physostigmine
- Administer dantrolene
The correct answer is:
- Remove the scopolamine patch placed preoperatively
This is likely scopolamine toxicity resulting in an anticholinergic toxidrome that presents with tachycardia, hyperthermia, hallucinations, and dry membranes.
Removal of the patch usually alleviates the symptoms of scopolamine toxicity and is typically done first.
Certain populations are at an increased risk of scopolamine toxicity. Those include older patients, pregnant, children, and patients with liver and kidney failure.
Physostigmine 1 to 4 mg IV can be used in cases where toxidrome does not resolve with patch removal.
A 78-year-old female with a recent diagnosis of diabetes mellitus was scheduled for an open reduction internal fixation of her left femoral fracture. Induction was performed using 80 mg propofol and 50 mcg fentanyl with subsequent 6mg vecuronium. After intubation, anesthesia was maintained with 1% - 3% sevoflurane. Up to this point, hemodynamics have remained stable. Three minutes after intubation, the patient’s heart rate slowed from 80 beats per minute to 55 beats per minute with a simultaneous decrease in blood pressure. Which of the following is most likely related to the inability to maintain appropriate cardiac output?
- Female gender
- Age greater than 65 years old
- Diabetes mellitus
- Osteoporosis
The correct answer is:
- Age greater than 65 years old
In young and healthy patients, the cardiac output is frequently preserved through compensatory mechanisms. These mechanisms are diminished with increased age. As a result, the cardiac output is not preserved during maintenance anesthesia, and both heart rate and blood pressure are diminished.
Elderly patients have multiple changes to the cardiovascular system that alter the response to inhaled anesthetics. These changes include stiffening of connective tissues and altered response to beta-receptor stimulation. As a result, the cardiac output is often not preserved during maintenance anesthesia.
Inhalation anesthetics have been shown to decrease diastolic function in the elderly by decreasing myocardial relaxation. This is in addition to the reduced myocardial contractility and stiffening of vascular structures.
Elderly patients with a history of diminished cardiac function have a greater reduction of blood pressure and cardiac index during the administration of inhaled anesthetics.
A 21-year-old female is undergoing an appendectomy. The client has a pulse of 110 beats per minute and a blood pressure of 100/60 mm of Hg. The provider uses ketamine for induction during the appendectomy. What adverse effect of this particular agent usually requires prophylactic treatment?
- Hypotension
- Bronchoconstriction
- Delusion
- Nausea and vomiting
The correct answer is:
- Nausea and vomiting
Nausea and vomiting are the most common side effects of ketamine.
Often a prophylactic agent is used for clients planned to receive ketamine.
Ketamine is often used in children and adolescents, especially with low blood pressure as it can elevate blood pressure. Ketamine is approved as a pre-anesthetic to other general anesthetic agents.
Delusions and bronchoconstriction are not seen with ketamine. The use of ketamine has often been associated with dissociation, emergence delirium, hallucinations, and bronchodilation.
Despite double gloving, a provider sustained a needle stick injury assisting in a procedure on a patient with HIV. After local care, how soon should post-exposure prophylaxis be started?
- Within 24 hours
- Only when symptoms develop
- After results of blood tests are available
- Only if the needle stick was deep
The correct answer is:
- Within 24 hours
If a needle stick injury is confirmed, one must start post-exposure prophylaxis (PEP) within the first 24 hours.
After any needlestick injury, the affected body part must be washed with water and soap immediately.
There is no evidence indicating that washing with an antiseptic reduces the rate of transmission.
Needlestick injuries create an average risk for HIV transmission after percutaneous exposure to HIV-infected blood of 3 per 1000 injuries or 1 per 1000 after mucocutaneous exposure.
A 27-year-old laboring patient at 38 weeks of gestation is undergoing epidural catheter placement for pain management. The baseline heart rate is 80bpm and blood pressure 110/70mmhg. After localization of the epidural space with loss of resistance technique, a combination of lidocaine 2% and epinephrine 1:200,000 is pushed through the catheter. Immediately after, the heart rate increases to 95bpm, and the patient complains of ringing in her ears, metallic taste in her mouth, and tingling around her lips. What mechanism explains the current presentation of this patient?
- Drug toxicity
- Intravascular catheter placement
- Local anesthetic reaching cranial nerves
- Expected side effects of epidural anesthesia
The correct answer is:
- Intravascular catheter placement
The initial symptoms of local anesthetics toxicity include tinnitus, dizziness, circumoral numbness, tongue paresthesias, and blurred vision. This patient’s symptoms are consistent with the intravascular injection of the test dose.
The purpose of the test dose is to confirm that the catheter is correctly placed in the epidural space or if it has been placed intravascularly. When the catheter is placed intravascularly, the boluses or infusion of local anesthetic and/or opioids are injected into the systemic circulation — the symptoms presented by this patient evidence an intravascular placement of the catheter that needs correction.
Total spinal symptoms include fixed dilated pupils, aphonia, and loss of consciousness. Total spinal happens when local anesthetic diffuses in the arachnoid space and reaches the cranial nerves. This patient’s symptoms are caused by local anesthetic and epinephrine injected intravascularly.
The medications used at this point for the patient are consistent with the test dose and not with analgesia for labor. Some of the expected side effects of epidural anesthesia after correctly placed catheters include nausea, legs heaviness, and itching. This patient’s presentation suggests intravascular placement of the epidural catheter.
A 50-year-old female is admitted after elective open abdominal surgery. On the third postoperative day, she develops fever to 37.9 C and a dry cough. Respiratory rate and heart rate are within normal limits. Lung examination reveals clear breath sounds, with a slight dullness to percussion posteriorly over the lower lobes. The wound is clear and not erythematous. There is no tenderness or swelling of the lower extremities. Chest X-ray shows small bilateral basilar infiltrates. Three sets of two bottles for blood cultures are drawn. One bottle is positive for Gram-positive cocci in clusters, and five are negative. What is the next best step of management?
- Intravenous low molecular weight heparin
- Intravenous antibiotics
- Incentive spirometry
- Bronchoalveolar lavage for bacterial culture
The correct answer is:
- Incentive spirometry
Atelectasis is bronchial obstruction due to the collapse of lung airways.
Atelectasis is often found in admitted patients, especially those who have undergone intubation, long surgeries, and are in a prolonged state of lying supine.
Symptoms of a dry cough, shortness of breath, and fever typically occur within the first to third days after surgery.
Atelectasis should be differentiated from pneumonia, which often occurs at the third postoperative day, by the clinical signs of abnormal breath sounds and x-ray findings such as lobar consolidation or large infiltrates. Pulmonary embolism can show clinical signs of tachypnea, tachycardia, chest pain, and a normal chest x-ray. It should be evaluated with a CT angiogram and be treated with IV heparin. This patient most likely does not have bacteremia due to the likelihood of the positive blood culture being skin contamination caused by a gram-positive organism such as Staphylococcus epidermidis and does not require immediate treatment.
A postoperative patient has received intrathecal morphine. Which of the following is appropriate?
- Notifying the provider if the patient’s respiratory rate falls below 14 breaths/min
- Instructing the patient to avoid coughing and deep breathing for 24 hours
- Keeping flumazenil available
- Monitoring the respiratory rate and pattern at least hourly for 24 hours
The correct answer is:
- Monitoring the respiratory rate and pattern at least hourly for 24 hours
Patients should have their respiratory rate and pattern monitored during the administration of intrathecal morphine, as respiratory depression can occur.
Morphine sulfate is an opioid analgesic and may be delivered into the intrathecal space (spinal canal) for a 24-hour duration of action. Respiratory rates less than 12 breaths/min are considered reportable.
Coughing and deep breathing are encouraged in the postoperative patient to prevent pneumonia and atelectasis.
Flumazenil is the agonist (has an affinity for another drug’s cellular receptor sites) for benzodiazepines, not opioid analgesics such as morphine.
A 5-year-old boy is scheduled for adenoidectomy. He was admitted to the hospital last night and scheduled to undergo procedure around 12 pm. He has been nil-per-orum (NPO) since 4 am. Around 8 am, the patient becomes restless due to hunger and starts throwing temper tantrums. He is crying uncontrollably and refuses to cooperate with any preoperative assessment until he is fed. What is the next best step in the management of this patient?
- Observe strict NPO until the procedure time
- Administer oral midazolam
- Offer the boy some apple juice
- Postpone the procedure to the next available date
The correct answer is:
- Offer the boy some apple juice
The incidence of aspiration in both the adult and pediatric patient populations is very small. Nonetheless, preoperative fasting must be observed whenever possible. NPO guidelines state that no clear liquids should be ingested for 2 hours before surgery; no breast milk for 4 hours before surgery; no infant formula or light snack for 6 hours before surgery; and no solids (fatty or fried foods) for 8 hours before surgery.
Midazolam has a reliable amnestic affecting anterograde memory with our affecting retrograde memory. The primary objectives of the medication are to relieve anxiety and produce amnesia. However, the use of safe and effective non-pharmacologic approaches are preferred.
In certain circumstances, exceptions can be made for a hungry child who is crying uncontrollably. In this case, offering apple juice up to two hours before a procedure may calm the child.
Considerable effort goes into scheduling a procedure. It would be inadvisable to reschedule a procedure in this instance. Instead, the application of non-pharmacologic and pharmacologic interventions would be more appropriate.
A 16-year-old male student presents to the emergency department after being assaulted by a drug dealer. The patient is conscious and gives a history of being stabbed in the abdomen. The primary survey reveals a 3cm linear deep wound in his epigastric region. His vitals are taken that show a heart rate of 118 beats/minute, a blood pressure of 85/55 mmHg, and a respiratory rate of 20/minute. The patient is resuscitated and shifted to operation room for urgent exploratory laparotomy. During the laparotomy, an anterior gastric perforation is visualized, which is then repaired using the Modified Graham patch repair technique. What is the method of choice for managing postoperative pain in this patient?
- As needed pain medications on demand
- Patient-controlled analgesia
- Epidural anesthesia
- Spinal anesthesia
The correct answer is:
- Patient-controlled analgesia
In the hospital, patient-controlled analgesia (PCA) is the best way to deliver pain medication post-operatively.
An infusion pump delivers an intravenous analgesic to the patient when the patient pushes a button.
PCA reduces the wait-time to receive pain medication, reduces nursing work, and reduces the chance of medication errors.
PCA is useful for patients who have good mentation and are physically able to push the button.
A 45-year-old man presents with a history of papillary thyroid cancer undergoes general anesthesia during surgery for removal of malignant tissue. The patient has no comorbid conditions or contraindications for anesthesia. During the procedure, he experiences changes from the onset of automatic respirations to eventual respiratory paralysis. Based on this response, what stage of anesthesia is the patient most likely experiencing?
- Stage 1
- Stage 2
- Stage 3
- Stage 4
The correct answer is:
- Stage 3
Stage 3 occurs from the onset of automatic respirations to respiratory paralysis.
Stage 1 is typically described as the “induction stage.” Patients are sedated, but conversational. Breathing is slow and regular.
Stage 2 is marked by such features as disinhibition, delirium, uncontrolled movements, loss of lash reflex, hypertension, and tachycardia. Airway reflexes remain intact during this phase and can be hypersensitive to stimulation.
Stage 4 is overdose. This occurs when too much anesthetic medication is given relative to the amount of surgical stimulation and the patient has severe brainstem or medullary depression.
A 17-year-old male is brought to the hospital after a fall. He is being treated with chemotherapy for leukemia. His vital signs are within normal limits. His physical exam shows a 0.5 cm linear laceration on the dorsal aspect of his left arm, which is repaired with 2-octyl cyanoacrylate. Which of the following best describes the homecare instructions for this patient?
- Apply over-the-counter topical antibiotic ointment to the area twice a day
- Gently wash away any crusted blood from the area
- Allow extended bathing
- Return to the emergency department immediately if there is redness, swelling, or drainage from the area
The correct answer is:
- Return to the emergency department immediately if there is redness, swelling, or drainage from the area
Two octyl cyanoacrylate has inherent antibiotic properties, and topical antibiotics may even damage the integrity of 2 octyl cyanoacrylate.
While reaching maximum bonding strength quickly, friction such as scrubbing can damage the 2-octyl cyanoacrylate and is not recommended.
Showering or non-sustained water exposure is not a contraindication for 2-octyl cyanoacrylate; however, prolonged exposure such as swimming is not recommended.
While there have been studies that show no difference in infection rates between suture closure and 2-octyl cyanoacrylate, signs of infection such as redness, swelling, or pustular drainage should prompt presentation to medical attention.
A 30-year-old woman is brought to the emergency department after suffering a motor vehicle accident. The patient sustained several injuries on her lower extremities. Her head and trunk did not get hurt, and she did not lose consciousness. The patient has no significant medical history and takes no medications. She does not smoke or drink. Blood pressure is 135/90 mm Hg, the pulse is 108/min, respirations are 26/min, and the temperature is 99.3F. Cardiopulmonary and abdominal examinations are normal. Upper limbs have no abnormalities and show 5/5 strength in muscle and 2+ deep tendon reflexes. Her lower limbs show many bruises, lacerations, and deformities. X-rays show many lower extremity fractures on both sides. After an extensive evaluation and patient stabilization, she is taken for surgical fixation of the fractures. Succinylcholine is used for rapid-sequence intubation. Which of the following complications is most likely to occur in this patient?
- Hypotension
- Arrhythmia
- Acute renal failure
- Acute liver failure
The correct answer is:
- Arrhythmia
Succinylcholine is used for rapid sequence intubation, due to it rapid onset and offset of action, which makes its use favorable.
In patients with burns, severe muscle injury, or neuromuscular disease, succinylcholine can lead to a massive release of potassium from skeletal muscles and cause arrhythmias or cardiac arrest.
Other conditions sensitive to succinylcholine include stroke, denervation or disuse atrophy of a muscle, drowning, and tetanus. Other side effects of succinylcholine include malignant hyperthermia.
Hyperkalemia does not cause adverse effects until it reaches 6.5 meq/l. Halothane causes acute liver failure, and propofol causes severe hypotension.
A 65-year-old male presents to the emergency department with acute onset generalized abdominal pain, nausea, and vomiting. On examination, there is marked abdominal rigidity and guarding. No bowel sounds are audible. After a detailed evaluation, he was diagnosed with intestinal perforation. He is being prepped for surgery, and the nurse administers an intravenous antibiotic before the procedure. What is the role of administering this antibiotic?
- Treat underlying infection
- Maintain the serum antibiotic level until the surgery is over
- Decrease the risk of infection after surgery
- Protect the patient in the event of an aseptic technique lapse
The correct answer is:
- Decrease the risk of infection after surgery
Preoperative antibiotic prophylaxis is defined as administering antibiotics prior to performing surgery to help decrease the risk of postoperative infections.
The routine administration of prophylactic antibiotics is standard in surgeries in which extensive dissections and higher amounts of anticipated blood loss are expected.
In most surgeries, the intent is to ensure the bactericidal concentration has been reached in the blood and tissues before incision. In this case, the patient already has bacterial contamination of the peritoneum due to the perforation.
Cephalosporins are most commonly used. Other commonly used antibiotics include vancomycin, metronidazole, aminoglycosides, and clindamycin.
A 21 yr old male patient has been prescribed an anesthetic medication prior to a surgical procedure that resulted in the patient having visual distortions and feeling as is he is floating and detached from reality. After the patient begins regaining consciousness, he complains of nausea, vomiting and appears confused. The mechanism of action of the anesthetic that was prescribed relates to which of the following receptors?
- GABA
- NMDA
- Glycine
- Guanylyl cyclase
The correct answer is:
- NMDA
Ketamine is a dissociative anesthetic that works by blocking glutamic acid N-methyl-D-aspartate(NMDA) receptors.
Distortion of reality and nightmares are common with ketamine administration.
Ketamine is contraindicated in patients with schizophrenia.
Ketamine is popular as the “go-to” drug for moderate sedation/procedures. Side effects commonly seen with ketamine include nausea, vomiting, and re-emergence delirium. Desflurane acts by working on GABA receptors and glutamate receptors. Isoflurane binds to GABA, glutamate and glycine receptors. Guanylyl cyclase is recognized as the most sensitive physiologic receptor for nitric oxide.
A 65-year-old man is scheduled for an elective cholecystectomy the next day. The client has a past medical history of insulin-dependent diabetes mellitus. He is ordered to be nil per oral (NPO) after midnight and is scheduled to receive long-acting insulin before midnight. Which of the following is the next most appropriate course of action?
- Hold the insulin
- Administer insulin
- Call the attending and confirm the order
- Inform the attending after administering insulin
The correct answer is:
- Call the attending and confirm the order
Insulin should not be administered as the patient will not eat anything after midnight and may experience hypoglycemia.
The nurse should call the attending before administering insulin to confirm the order.
Patients who are on home insulin therapy should reduce the dose of long-acting basal insulin (glargine, detemir) by 20 to 25% the evening before surgery.
Patients with diabetes should preferably be scheduled for surgery early in the day. It is recommended to check the blood glucose in the preoperative area.
Which of the following antiarrhythmics is the best initial treatment for supraventricular tachycardia?
- Propranolol
- Procainamide
- Lidocaine
- Adenosine
The correct answer is:
- Adenosine
Adenosine is considered the drug of choice for acute supraventricular tachycardia.
Adenosine interacts with alpha-1 receptors on the surface of cardiac cells. If adenosine is ineffective, verapamil or diltiazem can be used.
Lidocaine is used intravenously for the treatment of ventricular arrhythmias. Propranolol is a beta-blocker used to treat high blood pressure, chest pain, and atrial fibrillation.
Procainamide is indicated to immediately relieve life-threatening ventricular arrhythmias, such as sustained ventricular tachycardia.
A middle-aged female is scheduled for a procedure on the elective list. During her preoperative assessment, the clinician decides to proceed with a Bier block for the procedure. What does this block provide?
- Anesthesia below the diaphragm
- Anesthesia to a distal portion of an extremity
- Anesthetic block surrounding a peripheral nerve
- Anesthetic block to a nerve group
The correct answer is:
- Anesthesia to a distal portion of an extremity
A Bier block is essentially injecting local anesthetic solutions into the venous system of an upper or lower extremity.
The extremity is exsanguinated by compression or gravity and isolated utilizing a tourniquet from the central circulation.
The Bier block is a safe, effective, and cost-effective way to provide anesthesia and analgesia for extremity surgeries of a short duration.
This elegant technique requires minimal additional equipment and can be performed in a variety of clinical environments.
A 53-year-old male patient with extreme obesity presents for a laparoscopic bowel resection. He is a Mallampati class IV with a history of difficult intubation. Which of the following drugs would be of greatest benefit during the planned fiberoptic intubation?
- Hydromorphone
- Epinephrine
- Ephedrine
- Glycopyrrolate
The correct answer is:
- Glycopyrrolate
A patient with extreme obesity is at increased risk of aspiration during induction. Opioids should be used sparingly with supplemental oxygenation to ensure responsiveness.
Vasoconstriction would not affect the visualization of the pharynx for fiberoptic intubation.
While cardiac output, contractility, and systemic vascular resistance would be increased to potentially help a deficiency of the patient during surgery, these do not aid in visualization of the vocal cords in a patient with extreme obesity.
Glycopyrrolate is useful because it improves visualization by drying secretions. It also increases the effectiveness of the topical anesthesia and helps blunt airway responses. This is the best choice among all the options listed.
After removal of a nasogastric tube, what is the next most common step?
- Oral hygiene
- Obtain an x-ray
- Administer food
- Irrigate the nose
The correct answer is:
- Oral hygiene
Oral hygiene should be given after NG tube removal.
Asymptomatic patients do not need an x-ray.
Most patients do not eat immediately after the removal of an NG tube.
There would be no reason to irrigate the nose.
A 24-year-old construction worker presents to the emergency department after a fall 30 minutes before arrival. He fell on some broken glass and sustained a laceration to his left forearm. 1% lidocaine is combined with an adjunctive medication to anesthetize the wound and reduce bleeding before exploring the injury. What is the mechanism of action of this adjunctive medication?
- Local vasodilation
- Local vasoconstriction
- Blocks sodium channels
- Blocks potassium channels
The correct answer is:
- Local vasoconstriction
Lidocaine has intrinsic vasodilatory effects, increasing local blood flow when injected into a wound. Adding low-dose epinephrine counteracts this by causing mild vasoconstriction, allowing anesthesia to be achieved while minimizing blood loss.
The vasoconstrictor effects of epinephrine also reduce the rate at which lidocaine is removed from the site of the procedure through the vasculature, meaning that anesthesia should be effective for longer.
When given with epinephrine, the maximum safe dose for lidocaine increases from 3 mg/kg to 7 mg/kg.
Systemic exposure to excessive lidocaine results in the central nervous system (CNS) and cardiovascular effects. CNS effects occur at lower blood plasma concentrations, and additional cardiovascular effects present at higher concentrations, though cardiovascular collapse may still occur with low concentrations.
A 6-month-old infant is brought to the operating room for pyloromyotomy. His mother reports that she applied a local anesthetic (2.5% lidocaine and 2.5% prilocaine) cream for his peripheral IV two hours ago. On exam, the child has a bluish tinge to his lips and fingertips and a measured oxygen saturation of 86% on room air. Supplemental oxygen is provided, the lidocaine 2.5% and prilocaine 2.5% cream is removed, and blood for lab analysis is sent. The lab reports a methemoglobin concentration of 23.6%. Concurrent use of which of the following medications is most likely responsible for this patient’s current condition?
- Penicillin
- Cefazolin
- Trimethoprim-sulfamethoxazole
- Clindamycin
The correct answer is:
- Trimethoprim-sulfamethoxazole
A eutectic mixture of 2.5% lidocaine and 2.5% prilocaine is used as a topical anesthetic for procedures ranging from lab draws and peripheral IV starts to laser treatment of port-wine stains and lumbar punctures.
Prilocaine can oxidize normal hemoglobin to methemoglobin, especially in neonates and infants who may have reduced levels of methemoglobin reductase.
Medications known to increase the risk of methemoglobinemia include, but are not limited to, sulfonamides, phenytoin, chloroquine, nitroprusside, phenobarbital, and acetaminophen.
Trimethoprim-sulfamethoxazole is a sulfonamide antibiotic used to treat otitis media and other bacterial infections. Methemoglobinemia is treated with supplemental oxygenation and IV fluids along with IV methylene blue or ascorbic acid.
A 23-year-old otherwise healthy primigravid woman presents to the labor and delivery unit for a planned Cesarean section for breech presentation. The risks of spinal anesthesia, including headache, nausea, and vomiting, are discussed with the patient, and the plan is to proceed with spinal anesthesia. A medication with which of the following mechanisms of action is most appropriate to reduce post-anesthesia nausea in this patient?
- Inhibition of the muscarinic actions of acetylcholine on autonomic effectors innervated by postganglionic cholinergic nerves
- Antagonism of H1 receptors
- Antagonism of serotonin activity at D2 receptors in the chemoreceptor trigger zone
- Antagonism of the G-protein coupled neurokinin-1 receptor
The correct answer is:
- Inhibition of the muscarinic actions of acetylcholine on autonomic effectors innervated by postganglionic cholinergic nerves
Glycopyrrolate given immediately before spinal for an elective cesarean section can reduce the incidence and severity of nausea without adverse effects on neonatal APGAR scores since it does not cross the placenta compared to atropine.
Unopposed parasympathetic activity after sympathetic blockade causes increased peristalsis of the gastrointestinal tract, leads to nausea, and is the primary mechanism behind nausea after spinal. Atropine can be given for treating nausea after high spinal blockade but crosses the placenta and should be used with caution.
Nausea and vomiting can occur after spinal anesthesia in approximately 20% of patients. Risk factors include a high block (above T5 level), hypotension, opioid administration, and unopposed parasympathetic activity.
Hypotension can contribute to or even worsen nausea. Many providers give phenylephrine or ephedrine to treat nausea, but hypotension is not the primary mechanism of nausea after the spinal blockade.
A 45-year-old woman is scheduled for a diagnostic laparoscopy under general anesthesia. She has no significant past medical history. The patient admits to having milk and toast two hours before the scheduled procedure. Which of the following is the best course of action?
- Proceed with general anesthesia using a rapid sequence intubation
- Proceed with spinal anesthesia without sedation
- Wait two more hours and proceed with general anesthesia
- Wait four more hours and proceed with general anesthesia
The correct answer is:
- Wait four more hours and proceed with general anesthesia
The best course of action is to wait four more hours for a total of six hours of nothing by mouth status before proceeding with general, regional, or sedation for a procedure.
Milk and toast are considered light meals. Six hours is the minimum nothing by mouth status that is recommended. The provider must ensure there are no other factors that may delay gastric emptying.
Urgent surgeries should be done within 1 to 2 days but can wait a short time to stabilize the patient medically.
Proceeding with spinal anesthesia is not recommended as there remains the possibility of complications that would require conversion to general anesthesia.
A 76-year-old man who underwent femoral-popliteal bypass surgery becomes unresponsive in the postoperative care unit, and a pulse cannot be palpated. Emergency response is activated, and defibrillator pads are placed. An EKG shows ventricular tachycardia. What is the best next step in the management of this patient?
- Perform CPR for 2 minutes
- Provide a biphasic shock of 50 J
- Provide a biphasic shock of 150 J
- Provide a monophasic shock of 200 J
The correct answer is:
- Provide a biphasic shock of 150 J
Defibrillation should be immediately performed for pulseless ventricular tachycardia or fibrillation.
According to American Heart Association, an initial biphasic shock of 120 to 200 J should be administered for ventricular tachycardia.
Most defibrillators provide a biphasic shock. If using monophasic shock, the required energy is 360 J.
Cardioversion is indicated for unstable ventricular tachycardia. Signs of instability include hypotension, altered mental status, signs of shock, ischemic chest discomfort, and acute heart failure. The carotid pulse is palpable in unstable ventricular tachycardia.
A 67-year-old man is intubated following rapid sequence induction at the roadside after being involved in a vehicle crash. Emergency medical services immediately transport him to the nearest trauma center for surgery. What range of end-tidal C02 would indicate that successful intubation and ventilation has been achieved in this patient?
- 0-5 mmHg
- 10-20 mmHg
- 35-45 mmHg
- 65-75 mmHg
The correct answer is:
- 35-45 mmHg
A normal end-tidal carbon dioxide (CO2) level is 35-45 mmHg.
An undetectable CO2 level may be seen in cardiac arrest, equipment failures, if the circuit is disconnected, or in a failed airway.
CO2 levels above the normal range of 35-45 mmHg can be seen with type 2 respiratory failure, such as in decompensated chronic obstructive pulmonary disease (COPD)
Hyperventilation can lead to low CO2 levels below the normal range of 35-45 mm Hg.
In a post-open-heart surgery patient, excessive morphine has been administered. What is the most sensitive indicator of opioid-induced respiratory depression?
- Respiratory rate
- Level of consciousness
- Confusion
- Arterial CO2
The correct answer is:
- Level of consciousness
Drowsiness after opioid is often a marker for respiratory depression.
Before the respiration rate slows down, most people with opioid toxicity develop drowsiness or a decreased level of consciousness.
Several drugs have been developed which can block respiratory depression completely without affecting analgesia.
Once drowsiness occurs, apnea quickly follows. Naloxone can reverse this stage immediately.
A 27-year-old male in the postoperative ICU is complaining of difficulty breathing. He underwent an open reduction and internal fixation of the right femur two days ago. He was involved in a motor vehicle accident three days ago and suffered a displaced right femur fracture, three fractured ribs on the right side, a pulmonary contusion, a grade 1 liver laceration, and a 5 cm laceration on the forehead. He has a history of asthma for which he takes albuterol as needed. His vital demonstrate a blood pressure of 142/83 mmHg, heart rate 88/min, SpO2 97%, and respiratory rate 24/min. On examination, he is taking rapid, shallow breaths. His lungs are clear to auscultation bilaterally, and his heart rate is regular. Which of the following is the next best step in the management of this patient?
- Albuterol
- CT angiogram
- Blood transfusion
- Improved pain control
The correct answer is:
- Improved pain control
Inadequate pain control increases the likelihood of developing atelectasis by encouraging “splinting” - i.e. shallow breathing secondary to pain.
Additional factors that increase the risk for developing atelectasis include recent general anesthesia with abdominal and thoracic procedures increasing risk the most, obesity, pregnancy, high FiO2 during induction, and maintenance, and immobility.
Age, sex, and chronic lung disease do not increase the risk of developing atelectasis.
This patient is unlikely experiencing an asthma exacerbation as his lungs are clear to auscultation.
A 60-year-old man with abdominal pain and distention presents to the emergency room. He has vomited multiple times in the last two days. An emergency exploratory laparotomy is recommended. Which of the following is recommended to decrease the chances of aspiration during general anesthesia?
- Tablet famotidine PO 40 mg
- IV metoclopramide 10 mg
- Nasogastric decompression prior to induction
- Slow controlled induction of anesthesia
The correct answer is:
- Nasogastric decompression prior to induction
Metoclopramide is a prokinetic drug. It increased gastric emptying and small bowel peristalsis. Metoclopramide is contraindicated in patients with bowel obstruction due to the risk of bowel perforation.
Placement of a nasogastric tube in the awake patient before anesthesia and decompression of the stomach prior to induction of anesthesia is recommended in these patients.
Rapid sequence induction should be done in a patient at a high risk of aspiration. Slow controlled induction is recommended for patients with cardiac problems.
Famotidine is not likely to be effective in this emergency case. He has distention of the abdomen and needs decompression. He is also vomiting, so he may not tolerate oral famotidine.
A 65-year-old African American male with a past medical history of hypertension presents to the emergency department for confusion and abdominal pain. An emergent CT abdomen and pelvis was performed, which showed retroperitoneal pooling of the blood from possible abdominal aortic aneurysm rupture. He was transferred immediately for abdominal aortic rupture repair. He receives 9 L of Ringer’s lactate solution and four units of whole blood during the operation. Subsequently, a pulmonary artery catheter was placed, and he was transferred to the surgical intensive care for further management. Two hours after transfer to the surgical intensive care unit, the following hemodynamic parameters are obtained: systemic blood pressure (BP) 90/60 mm Hg, pulse 110 beats per minute, central venous pressure (CVP) 7 mm Hg, pulmonary artery pressure 28/10 mm Hg, pulmonary capillary wedge pressure (PCWP) 8 mm Hg, cardiac output 1.9 U/min, systemic vascular resistance 1400 (dyne - s)/cm5 (normal is 900 to 1300), Pa02 140 mm Hg, urine output 15 mL/h (specific gravity: 1.029), and hematocrit 35%. Given this data, which of the following is the most appropriate next step in management?
- Administration of a diuretic to increase urine output
- Administration of a vasopressor agent to increase systemic blood pressure
- Administration of a fluid challenge to increase urine output
- Administration of a vasodilating agent to decrease elevated systemic vascular resistance
The correct answer is:
- Administration of a fluid challenge to increase urine output
A ruptured abdominal aortic aneurysm is a surgical emergency often accompanied by severe hypotension and vascular collapse before surgery and massive fluid shifts with renal failure after surgery. In this case, all the hemodynamic parameters indicate inadequate intravascular volume, and the patient is, therefore, suffering from hypovolemic shock
The low urine output indicates poor renal perfusion, while the high urine specific gravity indicates adequate renal function with compensatory free-water conservation. The administration of a vasopressor agent would undoubtedly raise the blood pressure, but it would do so by increasing peripheral vascular resistance and thereby further decrease tissue perfusion.
The systemic hemodynamic readings signify the patient is in a hypovolemic state. Fluid resuscitation to expand the intravascular volume is the most appropriate next step in management. This can be quickly done with lactated Ringer’s solution or blood (or both) until improvements in such parameters as the PCWP, urine output, and blood pressure are noted.
Vasodilators are not appropriate at this time as the patient is already hypotensive and this would further worsen the patient’s condition.
An adult female is in the preoperative room awaiting colon surgery. What would be the best time for the client to receive an antibiotic to prevent a surgical infection?
- Two hours to twenty-four hours before surgery
- Less than 1 hour before surgery
- After the incision is made
- During the procedure
The correct answer is:
- Less than 1 hour before surgery
Surgical infections can be reduced by administering parenteral antibiotics less than 1 hour before making the incision.
Antibiotic choice is based on the likely pathogens that might be encountered.
There is no benefit seen in dosing prophylactic antibiotics 24 hours before the surgery.
However, antibiotic prophylaxis prior to dental surgery is now limited to a very small subset of the client population. According to the American Dental Association, the prophylactic medication should be limited to “patients with underlying cardiac issues associated with the highest risk of adverse outcome from infective endocarditis.”
A 30-year-old man undergoing shoulder surgery has been given an interscalene block. However, about 40 minutes after the block, the patient finds that his voice is getting hoarse, and he is finding it difficult to speak. Which of the following is the most likely cause?
- Phrenic nerve block
- Recurrent laryngeal nerve block
- Cervical sympathetic block
- Local anesthetic toxicity
The correct answer is:
- Recurrent laryngeal nerve block
The recurrent laryngeal nerve block is a well-known complication after interscalene block.
A recurrent laryngeal nerve block can cause hoarseness of voice due to ipsilateral paralysis of the vocal cord. The other mentioned complications are possible but are unlikely to be responsible for the change in voice.
Interscalene block is generally performed on patients undergoing shoulder, upper arm, or elbow surgery. It is not recommended for hand surgery as the inferior trunk may be spared.
This block is contraindicated in patients with respiratory insufficiency due to the high likelihood of ipsilateral phrenic nerve block and diaphragmatic hemiparesis. This can lead to a 25% reduction in pulmonary function. Additionally, the recurrent laryngeal nerve may be blocked, which could cause complete airway obstruction with anyone who has an existing vocal cord palsy.
A 28-year-old female at 38 weeks of gestation presents with labor pains. She has been in prolonged labor for over 16 hours. The obstetrician notes that the fetus shows mild signs of distress, and an urgent cesarean section is planned in the next hour. At what stage should the antibiotics be administered to have the maximal benefit in this patient?
- While the patient is still in labor on the delivery floor
- After clamping the cord
- At the time of anesthesia
- After the infant is delivered
The correct answer is:
- At the time of anesthesia
During a cesarean delivery, the endometrial cavity can be seeded with pathogens carried from the birth canal that put mothers at risk for endometritis and surgical-site infections. To reduce these risks, prophylactic administration of an intravenous periprocedural antibiotic, usually cefazolin, is routinely recommended. Such an infusion reduces these rates by about half.
A single dose of antibiotics administered before the rupture of membranes is considered most effective in women undergoing an elective cesarean section.
Unless there is a known infection, prophylactic antibiotics should be discontinued within 24 hours.
The current consensus is that the antibiotic should be administered before the cord is clamped.
A patient is scheduled for same-day surgery. The anesthesiologist plans to use a local anesthetic such as lidocaine. What is the rationale for including epinephrine with the anesthetic?
- Decreases anesthetic toxicity
- Prolongs anesthetic action
- Increases anesthetic solubility
- Increases anesthetic excretion
The correct answer is:
- Prolongs anesthetic action
Local anesthetics are membrane-stabilizing agents and reversibly decrease the rates of depolarization and repolarization.
Local anesthetics act mainly by inhibiting sodium influx in nerves, thus preventing the initiation of the action potential.
Local anesthetics are weak bases and are formulated with hydrochloric salt to make them water-soluble. Acidosis caused by inflammation reduces the action of local anesthetics.
Epinephrine often is added to a local anesthetic because it is a vasoconstrictor that prolongs the duration of action.
A 24-year-old G2P0A1 female is admitted to the labor room for delivery. Since there is cephalo-pelvic disproportion, she is being planned for an emergency cesarean section under spinal anesthesia. She is kept in the left lateral position, and the anesthetist prepares to administer the anesthetic agent. He asks specifically for bupivacaine anesthetic containing dextrose as an additive. For what reason would this be necessary?
- To provide nutrition for the spinal cord cells
- Higher density/baricity
- Reduce headache
- Reduce bleeding
The correct answer is:
- Higher density/baricity
The addition of additives like dextrose increases the baricity of local anesthetic solutions. The density determines where the solution lays relative to the cerebrospinal fluid. High-density solutions are hyperbaric. Solutions with the same density as CSF are isobaric. The solution can be made less dense than CSF by adding distilled water. Hyperbaric solutions settle at dependent areas; hence patient positioning is a factor in determining the level of blockade.
The spinal cord is perfused and nutrition is provided by the anterior and posterior spinal arteries, along with supply from radicular branches. Dextrose in spinal solutions has no role to play in the nutrition of spinal cord elements.
The incidence of post-dural puncture headache is not affected by the baricity of the solution.
Bleeding and baricity of solution have no correlation.
A 49-year-old man is scheduled for distal radius reduction and internal fixation. His past medical history is significant for hypertension and hyperlipidemia. A supraclavicular block is planned for postoperative pain control, performed in the preoperative holding area without sedation since the patient is very calm. Which of the following parameters is most critical to be monitored before performing this block in this patient?
- End-tidal CO2 monitoring
- EKG
- Temperature
- Urine output
The correct answer is:
- EKG
In 2011 the American society of anesthesiologists released updated guidelines for basic anesthetic monitoring, which specifies monitoring techniques for general anesthetics, regional anesthetics, and monitored anesthesia care.
The updated guidelines state, “every patient receiving anesthesia shall have the EKG continuously displayed from the beginning of anesthesia until preparing to leave the anesthetizing location.”
Temperature monitoring is not essential for the patient undergoing regional anesthesia without sedation.
Blood pressure should also monitored every five minutes.
A clinician is finishing a procedure, which was taking place under general anesthesia. The clinician is preparing the drug to reverse the muscle relaxants. Which of the following is most appropriate to be used to achieve this effect?
- Naloxone
- Protamine sulfate
- Neostigmine
- Diazepam
The correct answer is:
- Neostigmine
Neostigmine is water-soluble, an ionized compound that reversibly inhibits the enzyme acetylcholinesterase.
Its FDA indication is for the reversal of the effects of non-depolarizing neuromuscular blocking agents after surgery.
The use of neostigmine is primarily found in the reversal of neuromuscular blockade during the administration of anesthesia to patients undergoing surgery requiring muscle relaxation.
After administration of neostigmine, the concentration of acetylcholine is increased in the neuromuscular junction, which allows for muscles to contract with full strength and patients can breathe spontaneously and protect their airways safely after emergence from anesthesia.
A 10-year-old boy is transferred to the post-anesthesia care unit following a tonsillectomy. The nurse uses the Modified Aldrete score to assess the patient’s readiness for discharge. The patient should at least have which of the following scores for discharge?
- 6
- 7
- 8
- 9
The correct answer is:
- 9
The Modified Aldrete Score (MAS) can be used to assess a patient’s readiness for discharge.
The MAS assesses patient activity, respiration, blood pressure, consciousness, and color.
A patient with a MAS score of 9 out of 10 is considered adequate for discharge from the post-anesthesia care unit.
Before discharge, each patient should be thoroughly assessed for risk of future complications.
A 40-year-old woman with fevers, cough, and productive sputum for five days, is admitted to the intensive care unit requiring intubation and mechanical ventilation. She presented to the emergency department with acute respiratory distress syndrome (ARDS) secondary to pneumonia. Oxygenation saturation had been satisfactory on an FIO2 of 50% and positive end-expiratory pressure (PEEP) of 6 cm H2O but dropped to the low 80% despite an increase in FIO2 to 100%. On physical examination, the patient is intubated and sedated. Lung exam reveals diffuse rales and rhonchi. The examination of the heart is unremarkable except for tachycardia (heart rate of 115 beats/min). She has 2+ peripheral edema. A chest x-ray shows diffuse bilateral infiltrates. She is being ventilated using an assist/control mode with a tidal volume of 6 mL/kg and a plateau pressure of 25 cm H2O. Which of the following is the most appropriate strategy for PEEP in this patient?
- Increase PEEP in 2- to 3-cm H2O increments and lower FIO2 to at most 60%, if possible. Arterial oxygen saturation of at least 88% and 95% or less must be maintained
- Set the PEEP below the lower inflection point on a pressure-volume curve of the lung
- Maintain PEEP to correspond to the expiratory pressure that minimizes compliance of the lung
- Maintain PEEP of at least 14 cm H2O and increase up to 20 cm H2O for FIO2 of 0.5 to 0.8. Monitor cardiac output using a pulmonary artery catheter
The correct answer is:
- Increase PEEP in 2- to 3-cm H2O increments and lower FIO2 to at most 60%, if possible. Arterial oxygen saturation of at least 88% and 95% or less must be maintained
In this patient with ARDS secondary to pneumonia, increasing PEEP in 2- to 3-cm H2O increments and subsequently lowering FIO2 to at most 60% while maintaining an arterial oxygen saturation of at least 88% and 95% or less is appropriate.
This strategy has been used by ARDSnet and associated with improved outcomes. It is a “lung protective strategy”.
Lower PEEP is associated with reduced barotrauma.
There is no utility for a pulmonary artery catheter in the management of PEEP.
A 42-year-old woman at 37 weeks gestation presents to the hospital with vaginal bleeding. In her previous visits, she was told that she had gestational diabetes. Immediately she is taken to the labor room where a male fetus is delivered. After a short while, she starts experiencing sudden hypoxia, refractory hypotension, and hemorrhage. Unfortunately, she expires and the lung biopsy reveals fetal squamous epithelial cells. What is the gold standard for diagnosing this condition?
- Laboratory values that are consistent with disseminated intravascular coagulation (DIC)
- Chest X-ray consistent with acute respiratory distress syndrome (ARDS)
- Neurologic injury secondary to hypotension
- It is a diagnosis of exclusion
The correct answer is:
- It is a diagnosis of exclusion
Postpartum hemorrhage is the leading cause of maternal mortality and morbidity worldwide. Amniotic fluid embolism (AFE) represents the second leading cause of peripartum maternal death in the US and the number one cause of peripartum cardiac arrest. The presentation is abrupt, usually with sudden cardiorespiratory collapse followed by severe coagulopathy and refractory resuscitation. Survivors are frequently left with serious cardiac renal neurologic and pulmonary dysfunction.
The introduction of amniotic fluid and fetal components into the maternal circulation has been shown to create intense pulmonary vasoconstriction and bronco constriction. These effects are considered to be caused not only by physical obstruction but also by the liberation of intense inflammatory cytokines reactant to the foreign material. These inflammatory mediators are also believed to trip the coagulation and fibrinolytic pathways, creating a form of disseminated intravascular coagulation (DIC) syndrome.
Pulmonary wedge catheters (Swan Ganz) will directly measure left atrial and pulmonary artery pressures as well as cardiac output through thermal dilution. Pulmonary artery blood aspirants may reveal the presence of fetal components.
There is not a gold standard for diagnosis. Diagnosis relies on the clinical presentation, supporting laboratory evaluation and autopsy results, if necessary, and exclusion of other causes.
A 17-year-old male is on the operating table for a procedure when he suddenly develops a rash and bronchospasm after placement of a foley. What condition did the patient have that put him at higher risk for this reaction?
- Hypertension
- Congenital heart disease
- Diabetes
- Spina bifida
The correct answer is:
- Spina bifida
Rash and bronchospasm are the most common presenting symptoms for patients in the operating room with a latex allergy.
Epidemiologic studies have shown that a specific patient population such as those with spina bifida are at higher risk for latex allergy.
Those with numerous corrective surgeries and procedures as well as patients with repeated catheterization due to urological abnormalities are at increased risk.
Foley catheters are common sources of latex exposure in the hospital.
A 71-year-old with an acute myocardial infarction needs urgent surgery to repair his mitral valve. His blood pressure is low. Which of the following anesthetics would be very useful in this patient?
- Ketamine
- Succinylcholine
- Halothane
- Propofol
The correct answer is:
- Ketamine
Ketamine causes dissociative anesthesia and is a very useful analgesic agent.
Unlike most anesthetics, it does not cause depression of blood pressure and is useful in patients with a weak heart.
Ketamine can increase heart rate and systolic blood pressure during induction.
Ketamine does not depress respiration.
A middle-aged male patient is being prepared for a procedure. The procedure is expected to be short and requires no muscle relaxation. Which of the following inhalational agents is most likely to be used for this patient?
- Nitrous oxide
- Halothane
- Enflurane
- Isoflurane
The correct answer is:
- Nitrous oxide
Inhalation anesthetics are used for induction and maintenance of general anesthesia in the operating room.
Fundamentally, inhaled anesthetics work within the central nervous system by augmenting signals to chloride channels (GABA receptors) and potassium channels while depressing neurotransmission pathways.
Nitrous oxide, also known as “laughing gas,” is a non-flammable, odorless, and colorless gas. It has a minimum alveolar concentration of 104%. Unlike other inhaled anesthetics currently used, it has analgesic properties.
Nitrous oxide is favored for short procedures that may not require muscle relaxation as it has a speedy induction and recovery.